Showing that exponential functions are linearly independent

Click For Summary
To show that the functions e^(r_1t), e^(r_2t), and e^(r_3t) are linearly independent for distinct real numbers r_1, r_2, and r_3, the proof begins by assuming linear dependence and leads to a contradiction. By differentiating the equation c_1e^(r_1t) + c_2e^(r_2t) + c_3e^(r_3t) = 0, it is manipulated to yield e^((r_1 - r_2)t) = C e^((r_3 - r_2)t). Further manipulation shows that e^((r_1 - r_3)t) = C, leading to the conclusion that C must equal both 1 and a non-equal value, which is impossible. This contradiction confirms that the original assumption of linear dependence is false, thus proving the functions are indeed linearly independent. The argument hinges on the property that distinct exponential functions cannot be linearly dependent.
Mr Davis 97
Messages
1,461
Reaction score
44

Homework Statement


If ##r_1, r_2, r_3## are distinct real numbers, show that ##e^{r_1t}, e^{r_2t}, e^{r_3t}## are linearly independent.

Homework Equations

The Attempt at a Solution


By book starts off by assuming that the functions are linearly dependent, towards contradiction. So ##c_1e^{r_1t} + c_2e^{r_2t}+ c_2e^{r_3t} = 0##. After differentiating and doing some manipulations, the book finds that ##e^{(r_1 - r_2)t} = C e^{(r_3 - r_2)t}##, where C is just some constant. It then states that this is a contradiction, so the original statement must be true. I am, however, a little confused as to why this is a contradiction. Is it a contradiction based on some previously shown result that two exponential functions with different powers can never be linearly dependent?
 
Physics news on Phys.org
We need a few more steps to get a formal contradiction:

If we multiply both sides by ##e^{(r_2-r_3)t}## the equation becomes ##e^{(r_1-r_3)t}=C##. Substituting successively 0 and 1 for ##t## we get
$$1=e^{(r_1-r_3)\cdot 0}=C=e^{(r_1-r_3)\cdot 1}=e^{r_1-r_3}\neq 1$$
where the last inequality follows from the fact that ##r_1\neq r_3##.
 
  • Like
Likes Mr Davis 97
Question: A clock's minute hand has length 4 and its hour hand has length 3. What is the distance between the tips at the moment when it is increasing most rapidly?(Putnam Exam Question) Answer: Making assumption that both the hands moves at constant angular velocities, the answer is ## \sqrt{7} .## But don't you think this assumption is somewhat doubtful and wrong?

Similar threads

Replies
1
Views
1K
  • · Replies 3 ·
Replies
3
Views
2K
Replies
1
Views
2K
  • · Replies 4 ·
Replies
4
Views
2K
  • · Replies 6 ·
Replies
6
Views
3K
  • · Replies 2 ·
Replies
2
Views
1K
  • · Replies 2 ·
Replies
2
Views
2K
  • · Replies 28 ·
Replies
28
Views
5K
  • · Replies 2 ·
Replies
2
Views
2K
  • · Replies 2 ·
Replies
2
Views
2K